Questions

Réussis tes devoirs et examens dès maintenant avec Quizwiz!

The premium on a 3-year insurance policy expiring on December 31, Year 3, was paid in total on January 2, Year 1. If the company has a 6-month operating cycle, then on December 31, Year 1, the prepaid insurance reported as a current asset would be for:

Current items cover a period which is the company operating cycle (6 months) or a year, whichever is longer, and a year is longer than six months. Thus, for this company, items covering a 12-month period going forward are current.

Restorations of carrying value for long-lived assets are permitted if an asset's fair value increases subsequent to recording an impairment loss for which of the following? Both held for use and held for disposal Held for use Held for disposal Neither held for use nor held for disposal

A long-lived asset classified as held for sale (disposal) must be measured at the lower of its carrying amount or fair value less cost to sell. A loss should be recognized for any initial or subsequent write-down to fair value less cost to sell. A gain should be recognized for any subsequent increase in fair value less cost to sell, but not in excess of the cumulative loss previously recognized for a write-down to fair value less cost to sell. Held for disposal

During 20X1, Kam Co. began offering its goods to selected retailers on a consignment basis. The following information was derived from Kam's 20X1 accounting records: Beginning inventory $122,000 Purchases 540,000 Freight-in 10,000 Transportation to consignees 5,000 Freight-out 35,000 Ending inventory Held by Kam 145,000 Held by consignees 20,000 In its 20X1 income statement, what amount should Kam report as cost of goods sold? $507,000 $512,000 $527,000 $547,000

Cost of goods sold would include all inventoriable costs less ending inventory. Beginning inventory $122,000 Purchases 540,000 Freight-in 10,000 Cost of goods acquired 672,000 Add transportation to consignees 5,000 Cost of goods available $677,000 Less ending inventoryHeld by Kam $145,000 Held by consignees 20,000 165,000 Cost of goods sold $512,000

Topic 275 of the FASB's Accounting Standards Codification is entitled "Risks and Uncertainties." The primary subject discussed in this topic is: bankruptcy. going concern. disclosure. All of the answer choices are discussed.

One of the purposes of financial statements is to provide information to help users predict the reporting entity's future cash flows and results of operations. This assessment depends, to some degree, on the users' knowledge and assessment of the risks and uncertainties involving the entity's operations. Disclosure of these risks and uncertainties is a critical component of the user's process of evaluating these variables. FASB ASC 275-10 addresses the disclosures required to facilitate a user's evaluation of an entity's risks and uncertainties.

On January 1, 20X1, Bay Co. acquired a land lease for a 21-year period with no option to renew. The lease required Bay to construct a building in lieu of rent. The building, completed on January 1, 20X2, at a cost of $840,000, will be depreciated using the straight-line method. At the end of the lease, the building's estimated market value will be $420,000. What is the building's carrying amount in Bay's December 31, 20X2, balance sheet? $798,000 $800,000 $819,000 $820,000

This building is treated as a leasehold improvement. Although the land lease is for 21 years, the building will be in use for 20 years; thus, the depreciation period for the building is 20 years: Cost ÷ Life = $840,000 ÷ 20 years = $42,000/yearBuilding cost − Depreciation = Carrying value of building$840,000 − $42,000 = $798,000 Note: Leasehold improvements are depreciated fully without regard to salvage value if there is no option to renew (as is stated in this problem) and they are immovable (e.g., a building). In this case, because the land has a 21-year lease, any items permanently affixed to the land will revert to the owner at the end of the lease. Therefore, their market value at lease-end is irrelevant to Bay Co. because Bay Co. will no longer have the rights to the affixed property upon lease termination.

Which of the following is not reported in the income statement? Foreign currency translation gains Foreign currency remeasurement gains Foreign currency transaction gains All of the answer choices are reported in the income statement.

Foreign currency remeasurement gains and losses are reported in the income statement, as are foreign currency transaction gains and losses. Foreign currency translation gains and losses are reported in other comprehensive income.

On June 30, Huff Corp. issued at 99, 1,000 of its 8%, $1,000 bonds. The bonds were issued through an underwriter to whom Huff paid bond issue costs of $35,000. On June 30, Huff should report the bond liability at: $955,000. $990,000. $1,000,000. $1,025,000.

Accounting Standards Update (ASU) 2015-03 requires that debt issuance costs be presented in the balance sheet as a direct deduction from the carrying amount of the related debt liability, consistent with debt discounts; the recognition and measurement guidance for debt issuance costs were not affected by the amendments. Amortization of debt issuance costs also shall be reported as interest expense; issue costs will no longer be reported in the balance sheet as deferred charges. The carrying value of the debt, initially, the bond liability, is $990,000, computed as the number of bonds multiplied by the face amount per bond, multiplied by the issue percentage, reduced by the bond issue costs of $35,000: 1,000 bonds × $1,000 face × 0.99 = $990,000 $990,000 − $35,000 = $955,000

Which of the following items is included in accumulated other comprehensive income or loss? Unrealized gains and losses from the ineffective portion of a derivative properly designated as a cash flow hedge Unrealized holding gains or losses on investments in equity securities A reduction of shareholders' equity related to employee stock ownership plans Prior service costs not previously recognized as a component of net periodic pension costs

Accumulated other comprehensive income (AOCI) is a component of equity on the balance sheet, presented separately from retained earnings and additional paid-in-capital. Total items of other comprehensive income (OCI) are transferred to AOCI at the end of each reporting period. FASB defines OCI as "revenues, expenses, gains, and losses that under generally accepted accounting principles are included in comprehensive income but excluded from net income" (FASB ASC 220-10-20). These items have not yet been realized. Some examples of OCI include the following: Foreign currency translation gains or losses Gains and losses (effective portion only) on derivative instruments that qualify as cash flow hedges Unrealized holding gains and losses on available- for-sale debt securities Pension or postretirement gains or losses (not recognized immediately as a component of net periodic benefit cost) Prior service costs or credits Unrealized gains and losses from the ineffective portion of a derivative properly designated as a cash flow hedge and unrealized holding gains or losses on investments in equity securities are recognized immediately in income. A reduction of shareholders' equity related to employee stock ownership plans is generally the result of an increase in a related liability, not income.

An entity offers customers a mail-in rebate when the customer purchases the product of the entity. Which of the following statements is not correct? -An entity should recognize a refund liability if it receives consideration from a customer and expects to refund some or all of that consideration to the customer. -The refund liability and corresponding change in the transaction price should be updated at the end of each reporting period. -The entity would estimate the amount of rebate it expects to pay related to the rebate, reduce revenue recognized by that amount, and record a liability for the amount it expects to pay for the rebate. -The refund liability and the contract liability should be updated annually.

An entity should recognize a refund liability if it receives consideration from a customer and expects to refund some or all of that consideration to the customer. A refund liability is measured at the amount of consideration received or receivable for which the entity does not expect to be entitled; that is, amounts not included in the transaction price. The refund liability and corresponding change in the transaction price, and therefore the contract liability, should be updated at the end of each reporting period for changes in circumstances. The refund liability and the contract liability are not updated annually.

Which of the following factors would not be an indicator of an investor's ability to exercise significant influence over the operating and financial policies of an investee? -Investor recommendation for the investee to hire a specific executive -Interchange of managerial personnel between investor and investee -Investor representation on the investee board of directors -Dependence by the investee on the investor's proprietary technology

As a general rule, ownership of less than 20% (direct or indirect) of the voting stock of the investee leads to the presumption that an investor does not have the ability to exercise significant influence. This presumption can be overcome, however, if the ability to exercise significant influence can be demonstrated in other ways. Examples of such circumstances include the following: -Representation on the investee's board of directors -Participation in the investee's policy-making processes -Material intercompany transactions with the investee -Interchange of managerial personnel -Technological dependency of the investee on the investor Investor recommendation for the investee to hire a specific executive is not one of the listed circumstances. -Investor recommendation for the investee to hire a specific executive

Packet Corp. is in the process of preparing its financial statements for the year ended December 31, 20X1. How would a gain on remeasuring a foreign subsidiary's financial statements from the local currency into the functional currency that occurred during 20X1 be classified in these financial statements? Income from continuing operations, with no separate disclosure Income from continuing operations, with separate disclosure (either on the face of statement or in the notes) Other comprehensive income None of the answer choices are correct.

FASB ASC 830-30-45-17 provides: "It is also necessary to recognize currently in income all exchange gains and losses from remeasurement of monetary assets and liabilities." FASB ASC 830-30-45-18 states: "An analysis of the changes...shall be provided in a separate financial statement, in notes to the financial statements, or as a part of a statement of changes in equity." Income from continuing operations, with separate disclosure (either on the face of statement or in the notes)

In a period of rising general price levels, Pollard Corp. discloses income on a current cost basis. Which of the following contributes to Pollard's purchasing power loss on net monetary items? Refundable deposits with suppliers Equity investment in unconsolidated subsidiaries Warranty obligations Wages payable

Purchasing power gain or loss is computed by restating monetary assets and liabilities in units of constant purchasing power. Rising prices would cause liabilities to be paid with less valuable dollars, so equity investment in unconsolidated subsidiaries, warranty obligations, and wages payable would result in a purchasing power gain. Receipt of less valuable dollars from refundable deposits would result in a purchasing power loss.

Jonn City entered into a lease for equipment during the year. How should the asset obtained through the lease be reported in Jonn City's government-wide statement of net position? General capital asset Other financing use Expenditure Not reported

The GASB Codification clearly states that leased assets should be reported in government-wide financial statements. If used for general government purposes, the leased asset would be considered a general capital asset. Other financing use and expenditure are temporary accounts used in governmental funds to represent equity reductions.

The following information pertains to Ali Corp. as of and for the current year ended December 31: Liabilities $ 60,000 Stockholders' equity 500,000 Shares of common stock issued and outstanding 10,000 Net income 30,000 During the year, Ali's officers exercised stock options for 1,000 shares of stock at an option price of $8 per share. What was the effect of exercising the stock options? -Debt-to-equity ratio decreased to 12%. -Earnings per share increased by $0.33. -Asset turnover increased to 5.4%. -No ratios were affected.

The information presented is at the end of the year. The option exercise occurred during the year, resulting in these numbers. The ratio after the transaction is: $60,000 ÷ $500,000 = 0.12 (12%)

Cardamom Limited wants to get an idea about the effectiveness of its inventory management processes. The following data have been compiled to provide a benchmark for future comparisons. Net cash sales $ 51,000Cost of goods sold 272,000Inventory at beginning of year 37,000Purchases 263,000Accounts receivable at beginning of year 47,000Accounts receivable at end of year 52,000 What was the number of days' sales in average inventory turnover for the current year? 37.6 days 43.6 days 45.0 days 49.7 days

The number of days' sales in average inventory can be computed in at least two ways. The first way is to first compute inventory turnover and then divide that value into the number of days in the year. Inventory turnover is computed by dividing average inventory into cost of goods sold. Cost of goods sold is $272,000. Beginning inventory is $37,000. Ending inventory is the beginning inventory plus purchases, less cost of goods sold, and thus ending inventory is $28,000, computed as follows: $37,000 (Beginning inventory) + $263,000 (Purchases) - $272,000 (Cost of goods sold) = $28,000 The average of the beginning and ending inventory is $32,500, computed as follows: $37,000 (Beginning inventory) + $28,000 (Ending inventory) = $65,000 $65,000 ÷ 2 =$32,500; the inventory turnover is $272,000 ÷ $32,500, or 8.4 times. Finally, 365 days ÷ 8.4 = 43.5 days. It can also be computed as (Average Inventory ÷ Cost of Goods Sold) × Days per year: ($32,500 ÷ $272,000) × 365 = 43.6 days.

Crossroads Co. chooses to report a financial asset at its fair value. The asset trades in two different markets; however, neither market is the principal market for the financial asset. In the first market, sales proceeds are $76, which is net of transaction costs of $6. In the second market, the sales proceeds are $80, which is net of transaction costs of $1. What amount should Crossroads report as the fair value of the asset? $76 $80 $81 $82

In other words, in determining the most advantageous market, adjust for transaction costs. But to compute the actual fair value amount, ignore the transaction costs. The first market yields $76 and the second market yields $80, so the second market is the most advantageous. The fair value in the second market is $81 ($80 with the $1 added back. Operative word here is NET.

Mentor Co., a U.S. corporation, owned 100% of a Swiss corporation. The Swiss franc is the functional currency. The remeasurement of Mentor's financial statements resulted in a $25,000 gain at year-end. The translation of the financial statements resulted in a $40,000 gain at year-end. What amount should Mentor recognize as foreign currency gain in its income statement? $0 $25,000 $40,000 $65,000

Remeasurement gains and losses are reported directly on the income statement as foreign currency gains and losses, while translation gains and losses are recognized as a translation adjustment on the face of the balance sheet. Mentor would recognize the $25,000 remeasurement gain on the income statement. The $40,000 translation gain would appear on the balance sheet and would not impact Mentor's net income.

On January 1, 2X01, Big Oil placed in service an offshore oil platform that it constructed. Big Oil is legally required to dismantle and remove the platform at the end of its 10-year estimated life. Using expected present value techniques, Big Oil recorded an estimated asset retirement obligation (ARO) of $100,000 on January 1, 2X01. The ARO measurements on January 1, 2X01, are as follows: Expected cash flow before inflation: $190,000 Expected cash flow adjusted for inflation and market risk: $220,000 Present value using credit-adjusted risk-free rate: $100,000 Assuming that the ARO is settled on December 31, 2X10, for $170,000, what is the gain or loss on the settlement? $70,000 loss $20,000 gain $50,000 gain No gain or loss

The gain or loss on the settlement of the ARO liability is the difference between the ARO liability on settlement date of $220,000 and the actual settlement cost of $170,000. FASB ASC 410-20-40-2 requires the initial liability of $100,000 to be increased to the expected cash flow adjusted for market risk and inflation. Since the expected cash payment for the ARO liability was $220,000, a gain on settlement of $50,000 results. Note: Accounting for ARO liability is similar to the treatment of bonds payable. The liability is initially recorded at its present value and is amortized. Amortization is recorded with a debit to accretion expense and a credit to ARO liability.

Able Co. provides an incentive compensation plan under which its president receives a bonus equal to 10% of the corporation's income before income tax but after deduction of the bonus. If the tax rate is 40% and net income after bonus and income tax was $360,000, what was the amount of the bonus? $36,000 $60,000 $66,000 $90,000

To solve this problem, work backwards. If net income after the bonus and the taxes was $360,000, then (taking the tax expense back first) income before taxes was $600,000 (using the after-tax back to pre-tax conversion formula, $360,000 ÷ 0.6 = $600,000, 0.6 = 1 less the tax rate of 40%). Now, the bonus is equal to 10% of the income after deducting the bonus, which would be the income before taxes of $600,000. Thus the bonus is 0.10 × $600,000, or $60,000.

On January 1, year 1, a company capitalized $100,000 of costs for software that is to be sold. The company amortizes the software costs on a straight-line basis over five years. The carrying value of the software costs on January 1, year 3, was $60,000. As of December 31, year 3, the estimated future gross revenue to be generated from the sale of the software is $23,000, and the estimated future cost of disposing of the software is $8,000. What amount should the company expense related to the software costs for the year ended December 31, year 3? $18,400 $20,000 $37,000 $45,000

Software production costs are capitalized and reported at the lower of unamortized cost or net realizable value (NRV) once technological feasibility has been met. The unamortized cost is $60,000 and the NRV is $15,000 ($23,000 − $8,000); therefore, the software should be written down by $45,000 (i.e., expensed) to the NRV of $15,000.

Young Corp. purchased equipment by making a down payment of $4,000 and issuing a note payable for $18,000. A payment of $6,000 is to be made at the end of each year for three years. The applicable rate of interest is 8%. The present value of an ordinary annuity factor for three years at 8% is 2.58, and the present value for the future amount of a single sum of one dollar for three years at 8% is 0.79. Shipping charges for the equipment were $2,000, and installation charges were $3,500. What is the capitalized cost of the equipment? $19,480 $21,480 $24,980 $27,500

he capitalized cost of the equipment is $24,980: Down payment $ 4,000 Present value of note($6,000 × 2.58) 15,480 Shipping charges 2,000 Installation charges 3,500 Total $24,980

Wollongong Company decided to begin using dollar-value LIFO at the beginning of 20X5. The inventory value at January 1, 20X5, was $250,000. The current cost of the inventory at December 31, 20X5, was $306,000. At the end of 20X6, the current cost of the inventory was $288,750. The relevant index at the end of 20X5 was 1.02 and at the end of 20X6 was 1.05. The amounts Wollongong should report for inventory at the end of 20X5 and 20X6 are: $300,000 (20X5) and $275,500 (20X6). $301,000 (20X5) and $276,250 (20X6). $306,000 (20X5) and $276,250 (20X6). $301,000 (20X5) and $275,500 (20X6).

Adjusting 20X5 ending inventory results in cost at base-year prices of $300,000 ($306,000 ÷ 1.02). The $300,000 cost at base-year prices includes two layers: beginning of 20X5 layer = $250,000 and end of 20X5 layer = $50,000. The adjustment of the end of 20X5 layer to current-year prices = $50,000 × 1.02 = $51,000. $250,000 + $51,000 = $301,000 is reported for inventory at the end of 20X5. Adjusting 20X6 ending inventory results in cost at base-year prices of $275,000 ($288,750 ÷ 1.05). The $275,000 cost at base-year prices includes two layers: beginning of 20X5 layer = $250,000 and end of 20X5 layer = $25,000. Because the cost at base-year prices at the end of 20X6 is less than the cost at base-year prices at the end of 20X5 ($275,000 vs. $300,000), the layer created in 20X5 is reduced to $25,000 ($50,000 − $25,000). The adjustment of the end of 20X5 layer to current-year prices = $25,000 × 1.02 = $25,500. $250,000 + $25,500 = $275,500 is reported for inventory at the end of 20X6.

Baker Co. began its operations during the current year. The following is Baker's balance sheet at December 31: Baker Co.BALANCE SHEET Assets Cash $192,000 Accounts receivable 82,000 Total assets $274,000======== Liabilities and stockholders' equity Accounts payable $ 24,000 Common stock 200,000 Retained earnings 50,000 Total liabilities and stockholders' equity $274,000======== Baker's net income for the current year was $78,000 and dividends of $28,000 were declared and paid. Common stock was issued for $200,000. What amount should Baker report as cash provided by operating activities in its statement of cash flows for the current year? $20,000 $50,000 $192,000 $250,000

Baker should report $20,000 as net cash provided by operating activities: Net income $78,000 Adjustments Increase in accounts receivable ($82,000) Increase in accounts payable 24,000 (58,000) Net cash provided by operating activities $20,000

Ball Corp. had the following foreign currency transactions during the current year: Goods purchased from a foreign supplier on January 20 for the U.S. dollar equivalent of $90,000. The invoice was paid on March 20, at the U.S. dollar equivalent of $96,000. On July 1, Ball borrowed the U.S. dollar equivalent of $500,000 evidenced by a note that was payable in the lender's local currency on July 1, in two years. On December 31, the U.S. dollar equivalents of the principal amount and accrued interest were $520,000 and $26,000, respectively. Interest on the note is 10% per annum. In Ball's year-end income statement, what amount should be included as foreign exchange loss? $0 $6,000 $21,000 $27,000

Ball's year-end income statement should include $27,000 as foreign exchange loss, calculated as follows: Foreign currency loss on goods purchased ($90,000 - $96,000) $ 6,000Loan principal foreign currency loss ($500,000 - $520,000) 20,000Loan interest foreign currency loss ($25,000* - $26,000) 1,000Total loss $27,000 * Interest in U.S. dollars: $500,000 × 0.10 × 1/2 year = $25,000

A company reported the following for the current year: Retained earnings appropriated for plant expansion $32,500Correction of understated depreciation expense fromprior periods 9,300Unrealized loss on available-for-sale debt securities 8,100Unrealized gain on foreign currency translation 3,400 The company's current-year net income was $86,500, and the company has a 30% effective income tax rate. What amount of comprehensive income should be reported for the current year? $40,000 $76,700 $81,800 $83,210

Comprehensive income is net income +/− other comprehensive income (OCI) earned during the period. Items in other comprehensive income are reported net of tax. Comprehensive income is: Net income − Unrealized loss (1 - Tax rate) + Unrealized gain (1 - Tax rate) = Comprehensive income $86,500 − $8,100 (1 − 0.30) + $3,400 (1 − 0.30) = $83,210 Note: Retained earnings does not impact comprehensive income. Net income is closed to retain earnings at year-end. The correction of depreciation from prior periods would impact prior-period income and beginning retained earnings but would have no impact on current-year net income or OCI.

Which of the following should be disclosed as supplemental information in the statement of cash flows? Cash flow per share Conversion of debt to equity Both cash flow per share and conversion of debt to equity Neither cash flow per share nor conversion of debt to equity

FASB ASC 230-10-45-3 states very specifically that "financial statements shall not report an amount of cash flow per share." Also, "information about all investing and financing activities of an enterprise during a period that affects recognized assets or liabilities, but that does not result in cash receipts or cash payments in the period, shall be reported in related disclosures." (FASB ASC 230-10-50-3) Converting debt to equity is cited as an example of the latter category of items.

During Year 1, Wistrand Corporation purchased 12,000 shares of the 800,000 outstanding shares of Cherry Corporation's common stock for $79000. At the end of Year 1, Wistrand received $2480 of dividends from its investment in Cherry's stock. The fair value of Wistrand's investment on December 31, Year 1, is $93800. What amount of income or loss that is attributable to the Cherry stock investment should be reflected in Wistrand's earnings for Year 1? $2480 $3730 $14800 $17280

Equity securities representing less than 20% of the investee's outstanding common stock are marked to fair value at each reporting period, with gains and losses reported in earnings. Dividends are treated as dividend income. Therefore, there is both an increase from the dividend and from the increase in fair value that must be recognized: Dividend of $2480 + Increase in fair value ($93800 − $79000) of $14800 = Increase in income of $17280

Past events and current conditions potentially impacting the entity's future cash flow include which of the following? Subsequent events Segment reporting Related party transactions Timing of asset cash flows

Examples of past events and current conditions potentially impacting the entity's future line items and cash flows but which have not yet been incorporated into financial statement line items include existing or potential litigation; suspected or known statute, judicial, regulatory, or contract violations; unrecognized existing commitments expected to be recognized in the future; events where significant uncertainty led to the decision to not recognize the event; and subsequent events. Items not necessarily impacting line items that may require disclosure include dependency on one or a few customers or suppliers for profitability, input or output market volatility, uncertainty regarding an entity's access to markets for inputs or outputs or ability to maintain a qualified workforce, and other significant specific entity risk. Segment reporting and related party transactions are considered to be reporting entity disclosures, and the timing of asset cash flows is considered a financial statement line item explanation. Subsequent events

On December 31, 20X1, Jet Co. received two $10,000 notes receivable from customers in exchange for services rendered. On both notes, interest is calculated on the outstanding principal balance at the annual rate of 3% and payable at maturity. The note from Hart Corp. made under customary trade terms, is due in nine months and the note from Maxx, Inc., is due in five years. The market interest rate for similar notes on December 31, 20X1, was 8%. The compound interest factors to convert future values into present values at 8% follow: Present value of $1 due in nine months: .9440 Present value of $1 due in five years: .6806 At what amounts should these two notes receivable be reported in Jet's December 31, 20X1, balance sheet? Hart: $9,440; Maxx: $6,800 Hart: $9,652; Maxx: $7,827 Hart: $10,000; Maxx: $6,800 Hart: $10,000; Maxx: $7,827

FASB ASC 310 provides that notes receivable stating either no interest or an unreasonably low interest rate be reported at their present value computed using an appropriate interest rate if the original maturity date of the note exceeds one year. The Hart note would be reported at its face amount of $10,000 since it matures within the current 1-year accounting period. The correct value for reporting the Maxx note is: Present value of Maxx note= Maturity amount x Present value factor= ($10,000 + ($10,000 x 3% x 5 years)) x .6806= ($10,000 + $1,500) x .6806= $7,827 Hart: $10,000; Maxx: $7,827

During 20X1, Sloan, Inc., began a project to construct new corporate headquarters. Sloan purchased land with an existing building for $750,000. The land was valued at $700,000 and the building at $50,000. Sloan planned to demolish the building and construct a new office building on the site. What is the appropriate accounting treatment for interest of $186,000 on construction financing paid during construction? Classify as land and do not depreciate Classify as building and depreciate Expense None of the answer choices are correct.

FASB ASC 360-10-30-1 provides: "If an asset requires a period of time in which to carry out the activities necessary to bring it to that condition and location, the interest cost incurred during that period as a result of expenditures for the asset is a part of the historical cost of acquiring the asset." The $186,000 should be classified as building cost and would be depreciated. Classify as building and depreciate

Which of the following statements is correct concerning the appearance of noncontrolling interest on the income statement? -Revenues, expenses, gains, losses, net income or loss, and other comprehensive income are reported in the consolidated financial statements at the consolidated amounts, which include the amounts attributable to the owners of the parent and the noncontrolling interest. -Revenues, expenses, gains, losses, net income or loss, and other comprehensive income are reported in the consolidated financial statements as the amounts attributable to the owners of the parent, followed by a separate disclosure of the revenues, expenses, gains, losses, net income or loss, and other comprehensive income attributable to the noncontrolling interest. -Revenues, expenses, gains, losses, net income or loss, and other comprehensive income are reported in the consolidated financial statements at the owners' amounts with disclosure of the noncontrolling interest only in the footnotes. -None of the answer choices are appropriate disclosure of the noncontrolling interest on the income statement.

FASB ASC 810-10-45-19 requires that the consolidated amounts of these items (revenues, expenses, gains, losses, net income or loss, and other comprehensive income) be reported on the income statement. The amount of consolidated net income attributable to the parent and to the noncontrolling interest must be clearly identified and presented on the face of the consolidated statement of income. Revenues, expenses, gains, losses, net income or loss, and other comprehensive income are reported in the consolidated financial statements at the consolidated amounts, which include the amounts attributable to the owners of the parent and the noncontrolling interest.

Which of the following is not an eligible item for the fair value measurement option under FASB ASC 825-10-15-4? --A recognized financial asset and financial liability, except any listed below in exceptions -A firm commitment that would otherwise not be recognized at commencement and that involves only financial instruments (An example is a forward purchase contract for a loan that is not readily convertible to cash. That commitment involves only financial instruments—a loan and cash—and would not otherwise be recognized because it is not a derivative instrument.) -A written loan commitment -An interest in a variable interest entity that the entity is required to consolidate

FASB ASC 825-10-15-5 lists the following items that are not eligible for the fair value election: "An investment in a subsidiary that the entity is required to consolidate "An interest in a variable interest entity that the entity is required to consolidate "Employers' and plans' obligations (or assets representing net overfunded positions) for pension benefits, other postretirement benefits (including health care and life insurance benefits), postemployment benefits, employee stock option and stock purchase plans, and other forms of deferred compensation arrangements, as defined in [FASB ASC] Topics 420; 710; 712; 715; 718; and 960. "Financial assets and financial liabilities recognized under leases as defined in [FASB ASC] Subtopic 842-10 (This exception does not apply to a guarantee of a third-party lease obligation or a contingent obligation arising from a cancelled lease.) "Deposit liabilities, withdrawable on demand, of banks, savings and loan associations, credit unions, and other similar depository institutions "Financial instruments that are, in whole or in part, classified by the issuer as a component of shareholder's equity (including "temporary equity"). An example is a convertible debt security with a noncontingent beneficial conversion feature." -An interest in a variable interest entity that the entity is required to consolidate

In its December 31 balance sheet, Butler Co. reported trade accounts receivable of $250,000 and related allowance for uncollectible accounts of $20,000. What is the total amount of risk of accounting loss related to Butler's trade accounts receivable, and what amount of that risk is off-balance sheet risk? Risk of accounting loss: $0; Off-balance sheet risk: $0 Risk of accounting loss: $230,000; Off-balance sheet risk: $0 Risk of accounting loss: $230,000; Off-balance sheet risk: $20,000 Risk of accounting loss: $250,000; Off-balance sheet risk: $20,000

FASB ASC 825-10-50-20 defines risk of accounting loss as the amount of write-off that a company would record if any party to an agreement failed to fully perform in accordance with the terms of the contract. Off-balance sheet risk occurs when the amount of an accounting loss exceeds the amount of the associated asset or liability recorded on the balance sheet. The maximum possible accounting loss associated with trade accounts receivable occurs if no amount of the current asset is collected. In this case, Butler's trade accounts receivable has a net book value of $230,000, which represents the maximum amount of potential write-off associated with trade accounts receivable. Butler would not be required to pay an amount in addition to the net book value of this asset, so there is no off-balance sheet risk. Risk of accounting loss: $230,000; Off-balance sheet risk: $0

Paxton Co. signed contracts for the purchase of raw materials to be executed the following year at a firm price of $5 million. The market price of the materials dropped to $3 million on December 31. What amount should Paxton record as an estimated liability on purchase commitments as of December 31? $5,000,000 $3,000,000 $2,000,000 $0

Firm purchase commitments, situations when a company agrees to buy a fixed amount of inventory at a future date, are not recognized as an asset or liability on the commitment date. They are subject to remeasurement if the price of the asset falls below the agreed-upon firm price, creating a loss contingency. A contingency is an existing condition, situation, or set of circumstances involving uncertainty as to possible gain (gain contingency) or loss (loss contingency) to an enterprise that will ultimately be resolved when one or more future events occur or fail to occur. Proper accounting for loss contingencies requires an assessment of the probability (i.e., probable, reasonably possible, remote) that a future event or events will confirm a loss or asset impairment or the incurrence of a liability as of the date of the financial statements. As this loss is probable and the amount of the loss can be reasonably estimated, a $2,000,000 liability should be recognized as of December 31.

A company with a June 30 fiscal year-end entered into a $3,000,000 construction project on April 1 to be completed on September 30. The cumulative construction-in-progress balances at April 30, May 31, and June 30 were $500,000, $800,000, and $1,500,000, respectively. The interest rate on company debt used to finance the construction project was 5% from April 1 through June 30 and 6% from July 1 through September 30. Assuming that the asset is placed into service on October 1, what amount of interest should be capitalized to the project on June 30? $11,666 $18,750 $75,000 $90,000

For qualifying assets, the interest cost incurred during the period of construction of an asset is a part of the historical cost of acquiring the asset. In general, the capitalization rate shall be a weighted average of the rates applicable to borrowings (debt) outstanding during the accounting period for which the capitalizable interest is being calculated. Cumulative CWIP # of Months Capitalized Balance Interest Rate Outstanding Interest $ 500,000 5% 1/12 $ 2,083 800,000 5% 1/12 3,3331, 500,000 5% 1/12 6,250 TOTAL $11,666

On September 1, 20X1, Brady Corp. entered into a foreign exchange contract for speculative purposes by purchasing 50,000 deutsche marks for delivery in 60 days. The rates to exchange $1 for 1 deutsche mark follow: 09/01/X1 09/30/X1Spot rate .75 .7030-day forward rate .73 .7260-day forward rate .74 .73 In its September 30, 20X1, income statement, what amount should Brady report as foreign exchange loss? $2,500 $1,500 $1,000 $500

Foreign currency amount at 09/01/X1 ($.74 x 50,000) $37,000 Less: Foreign currency amount at 09/30/X1 ($.72 x 50,000) 36,000 Foreign exchange loss $ 1,000 =======

Campbell Corp. exchanged delivery trucks with Highway, Inc. Campbell's truck originally cost $23,000, its accumulated depreciation was $20,000, and its fair value was $5,000. Highway's truck originally cost $23,500, its accumulated depreciation was $19,900, and its fair value was $5,700. Campbell also paid Highway $700 in cash as part of the transaction. The transaction lacks commercial substance. What amount is the new book value for the truck Campbell received? $5,700 $5,000 $3,700 $3,000

Generally, a nonmonetary exchange should be based on the fair values of the assets exchanged—resulting in the immediate recognition of a gain or loss. Exceptions to this treatment include the following: Fair value is not determinable Exchange transaction to facilitate sales to customers Exchange transaction that lacks commercial substance Under these exceptions, no gains or losses are recognized. Since this transaction lacks commercial substance, no gain or loss is recognized and the new book value is equal to the book value prior to the exchange: Original cost $23,000 Accumulated depreciation 20,000 Book value $ 3,000 Additional cash paid 700 New book value $ 3,700

Amar Farms produced 300,000 pounds of cotton during the Year 1 season. Amar sells all of its cotton to Brye Co., which has agreed to purchase Amar's entire production at the prevailing market price. Recent legislation assures that the market price will not fall below $0.70 per pound during the next two years. Amar's costs of selling and distributing the cotton are immaterial and can be reasonably estimated. Amar reports its inventory at expected exit value. During Year 1, Amar sold and delivered to Brye 200,000 pounds at the market price of $.70. Amar sold the remaining 100,000 pounds during Year 2 at the market price of $0.72. What amount of revenue should Amar recognize in Year 1? $140,000 $144,000 $210,000 $216,000

Here the available produced finished inventory has a buyer already under contract obligation to buy, and a guaranteed minimum price, so revenue recognition upon production is appropriate. The inventory can be carried at net realizable value, and the sales price can be recognized as revenue in Year 1, upon production, of $210,000 (300,000 pounds × $0.70 a pound, the set minimum price for Year 1).

Which of the following statements related to initial direct costs (IDC) is correct? IDC include items such as commissions and payments made to current tenants to obtain the lease. IDC include all internal costs associated with obtaining the lease. IDC are included by the lessor in the measurement of an ROU asset. IDC are expensed as incurred.

IDC are capitalized (not expensed) and include only those costs that entity would not have incurred if the lease had not been entered into (i.e., must be incremental costs, not internal, such as commissions and payments made to current tenants to obtain the lease). The lessee (not the lessor) includes IDC in the measurement of a right-of-use (ROU) asset.

Drew Company leased equipment to Remy Inc. under a five-year finance lease agreement that includes a purchase option effective at the end of the lease term that is expected to be exercised. The equipment originally cost Drew $425,000 and has an expected economic life of 8 years and an expected residual value of $20,000 at the end of its economic life. Using the straight-line method, what would Remy record as annual amortization? $85,000 $81,000 $50,625 $53,125

If a purchase option is reasonably certain to be exercised, the lessee includes the present value of the exercise price in determining the right-of-use (ROU) asset and liability. However, payments are not given in this question, but rather the value of the asset and the residual value. Remy would record annual amortization of $50,625. The expected residual value of $20,000 is deducted from the cost of $425,000 and the remaining $405,000 is amortized over the eight-year economic life of the asset as it is expected that the purchase option will be exercised.

The following financial ratios and calculations were based on information from Kohl Co.'s financial statements for the current year: Accounts receivable turnover10 times during the year Total assets turnover 2 times during the year Average receivables during the year$200,000 What was Kohl's average total assets for the year? $2,000,000 $1,000,000 $400,000 $200,000

Incorrect One can use the ratios given along with some algebra to solve for average total assets. Accounts receivables turnover is the multiple of accounts receivable to get the sales total. The receivables turned over 10 times during the year, so the sales were 10 times the receivables, or $2,000,000 (10 × $200,000). Total asset turnover is the multiple of total assets to get sales. Since the total asset turnover is 2, the sales were twice the level of total assets, so dividing sales in half results in total assets of $1,000,000.

Larkin Co. reported a taxable loss of $10,000 in 20X1, its first year of operations, and taxable income of $0 in 20X2. Larkin had no temporary or permanent differences in either 20X1 or 20X2. At the end of 20X1 Larkin believed that 30% of the operating loss carryforward would not be realized; therefore, a valuation allowance of $1,200 (30% of $10,000 NOL × 40% tax rate) was necessary. At the end of 20X2, Larkin believes that the valuation allowance is no longer necessary. Assuming a tax rate of 40%, Larkin should report total income tax expense (benefit) in 20X1 and 20X2 of: $0 in 20X1 and $0 in 20X2. $(4,000) in 20X1 and $0 in 20X2. $(2,800) in 20X1 and $0 in 20X2. $(2,800) in 20X1 and $(1,200) in 20X2.

In 20X1, Larkin should recognize a noncurrent deferred tax asset and the related noncurrent deferred tax benefit of $4,000 ($10,000 NOL × 40% tax rate). However, Larkin also must recognize a $1,200 valuation allowance in 20X1. Thus, in 20X1 Larkin should recognize a net tax expense (benefit) of $(4,000) + $1,200 = $(2,800). Note that the recognition of the valuation allowance reduces the net tax benefit recognized in 20X1. The decision in 20X2 that the valuation allowance is no longer necessary means that the valuation allowance should be eliminated, as shown in the following entry in 20X2: Valuation allowance 1,200Tax expense/benefit - deferred 1,200 Therefore, tax expense (benefit) in 20X2 has a credit balance of $(1,200), indicating a deferred tax benefit. This $(1,200) tax benefit recognized in 20X2 is the change in deferred tax expense/benefit arising from changed circumstances causing a change in judgment as to the amount of valuation. $(2,800) in 20X1 and $(1,200) in 20X2.

On January 1, 20X1, Mollat Co. signed a 7-year lease for equipment having a 10-year economic life. The present value of the monthly lease payments equaled 80% of the equipment's fair value. The lease agreement provides for neither a transfer of title to Mollat nor a purchase option. In its 20X1 income statement, Mollat should report: rent expense equal to the 20X1 lease payments. rent expense equal to the 20X1 lease payments less interest expense. lease amortization equal to 1/10th of the equipment's fair value. lease amortization equal to 1/7th of 80% of the equipment's fair value.

In addition to transfer of title and purchase option, FASB ASC 842-10-25-2 provides three additional criteria for determining whether a lease is a finance lease: The lease term is for a major part of the economic life of the leased asset. The present value of the sum of the lease payments and any lessee guaranteed residual value not already in the lease payments equals or exceeds substantially all of the fair value of the underlying asset. The underlying asset is specialized and is not expected to have an alternative use to the lessor at the end of the lease term. FASB ASC 842-10-55-2 uses the former 75% and 90% rules as benchmarks. In Mollat's case the lease term is only 70% of economic life and the present value is only 80% of the equipment's fair value. The lease is, therefore, an operating lease and the lease payments should be reported as rent expense. rent expense equal to the 20X1 lease payments.

Ichor Co. reported equipment with an original cost of $379,000 and $344,000, and accumulated depreciation of $153,000 and $128,000, respectively, in its comparative financial statements for the years ending December 31, 20X2 and 20X1. During 20X2, Ichor purchased equipment costing $50,000, and sold equipment with a carrying value of $9,000. What amount should Ichor report as depreciation expense for 20X2? $19,000 $25,000 $31,000 $34,000

In the context of this problem accumulated depreciation is affected by the asset disposal when the carrying value of the asset sold is written off and by depreciation expense for the current period. These two items account for the net increase of $25,000 ($153,000 - $128,000) in the credit balance of the accumulated depreciation account. The debit change in accumulated depreciation caused by the asset disposal needs to be determined from the facts provided. The equipment account had a beginning balance of $344,000. The $50,000 purchase of new equipment would cause this balance to increase to $394,000. However, the ending balance was $379,000. The only other transaction affecting the equipment account was the disposal of a piece of equipment. Therefore, the original cost of the disposed equipment was $15,000 ($394,000 - $379,000). Since the disposed equipment had a cost of $15,000 and a carrying value of $9,000 (carrying value = cost - accumulated depreciation), the accumulated depreciation associated with the disposed equipment was $6,000 ($9,000 = $15,000 - accumulated depreciation). The beginning credit balance in the accumulated depreciation control account was $128,000. It would have been decreased (debited) for the $6,000 of accumulated depreciation related to the disposed equipment. That would leave a credit balance of $122,000. However, the ending balance was a credit of $153,000. Depreciation expense for the period would also change (increase or credit) the balance of accumulated depreciation. Since the ending balance was $153,000, and the balance without the effect of depreciation expense was $122,000, the depreciation expense must have been $31,000 ($153,000 - $122,000).

Orleans Co., a cash-basis taxpayer, prepares accrual-basis financial statements. In its current-year balance sheet, Orleans' deferred income tax liabilities increased compared to the previous year. Which of the following changes would cause this increase in deferred income tax liabilities? An increase in prepaid insurance An increase in rent receivable An increase in warranty obligations I only I and II II and III III only

Incorrect Deferred income tax liabilities are caused by items that defer payment of taxes, which cause more taxes to be paid in later years than the income tax expense taken currently. An increase in prepaid insurance can lower taxes now by adding to the expenses deductible, and cause deferral of taxes to the future, so it would qualify a change that would increase deferred tax liabilities. An increase in rent receivable, a pushing forward of the receipt of the rent in cash (when it will be taxed), can also defer taxes to the future and add to later taxes due, so it would also increase deferred tax liabilities. An increase in warranty obligations means one is pushing forward the paying of the expense in cash (which allows the deduction), and this would lower taxes in the future, not add to the future liabilities. All deferred tax assets and liabilities are classified as noncurrent on the balance sheet. I and II

Dodd Corp. is preparing its December 31 current-year financial statements and must determine the proper accounting treatment for the following situations: For the current year ended December 31, Dodd has a loss carryforward of $180,000 available to offset future taxable income. However, there are no temporary differences. Based on an analysis of both positive and negative evidence, Dodd has reason to believe it is more likely than not that the benefits of the entire loss carryforward will be realized within the carryforward period. On 12/31 of this year, Dodd received a $200,000 offer for its patent. Dodd's management is considering whether to sell the patent. The offer expires on 2/28 of next year. The patent has a carrying amount of $100,000 at 12/31. Assume a current and future income tax rate of 30%. In its current-year income statement, Dodd should recognize an increase in net income of: $0. $54,000. $70,000. $124,000.

Incorrect Deferred tax assets are measured by the total temporary differences multiplied by the tax rates in effect when the tax differences unwind. The loss carryforward is recognized as a deferred tax asset at the total future deductible amount multiplied by the future tax rate that will be available for the later tax deductions. All deferred tax liabilities and deferred tax assets are classified on the balance sheet as noncurrent. Thus, the deferred tax asset is a tax benefit (lowering of this year's income tax expense) and will increase net income by the total amount of the expected benefit amount of $54,000 ($180,000 deduction × 0.30 (the future tax rate of 30%)). The other gain is not recognized until the sale is finalized and agreed to by both parties.

Stam Co. incurred the following research and development project costs during the current year: Equipment purchased for current and future projects $100,000Equipment purchased for current projects only 200,000Research and development salaries for current projects 400,000Legal fees to obtain patent 50,000Material and labor costs for prototype product 600,000 The equipment has a 5-year useful life and is depreciated using the straight-line method. What amount should Stam recognize as research and development expense at year-end? $450,000 $1,000,000 $1,220,000 $1,350,000

Incorrect Research and development costs are identified in five categories: Materials, equipment, and facilities used in R&D activities Personnel engaged in R&D activities Intangibles purchased or developed for use in R&D activities Contract services acquired and used in conjunction with R&D activities Indirect costs reasonably allocable to R&D activities A particularly important aspect of determining whether items included in (1) and (3) above are research and development is whether the item (e.g., machinery, equipment, patent) has an alternative use in other R&D projects. If an alternative use exists, the cost is capitalized as a tangible or intangible asset and depreciated or amortized. The periodic depreciation or amortization is identified as R&D expense as long as the asset is used in R&D activity. If no alternative use exists, the expenditure is charged to R&D in the period of acquisition. Costs to obtain a patent are not R&D expenditures and should be capitalized. The research and development expenses include the following: Depreciation on equipment withan alternate use ($100,000 / 5) $ 20,000 Equipment for current project 200,000 Research and development salaries 400,000 Material and labor costs 600,000 Total $1,220,000

On January 2, 20X1, Lem Corp. bought machinery under a contract that required a down payment of $10,000, plus 24 monthly payments of $5,000 each, for total cash payments of $130,000. The cash equivalent price of the machinery was $110,000. The machinery has an estimated useful life of 10 years and estimated salvage value of $5,000. Lem uses straight-line depreciation. In its 20X1 income statement, what amount should Lem report as depreciation for the machinery? $10,500 $11,000 $12,500 $13,000

Incorrect The machinery is recorded at the cash equivalent price of the machinery, $110,000. Straight-line depreciation for 20X1 = $(110,000 - $5,000) / 10 years= $ 105,000 / 10 years= $ 10,500 The total cash payment of $130,000 includes interest of $20,000.

Sea Manufacturing Corp. is constructing a new factory building. During the current calendar year, Sea made the following payments to the construction company: January 2 $1,000,000 December 31 1,000,000 Sea has an 8%, three-year construction loan of $3,000,000. What is the amount of interest costs that Sea may capitalize during the current year? $0 $80,000 $160,000 $240,000

Interest expense is capitalized for assets that are constructed or otherwise produced for an enterprise's own use. The amount of interest to be capitalized during a given accounting period is determined by applying the appropriate capitalization rate to the average amount of accumulated expenditures for the asset during that period. In general, the capitalization rate is a weighted average of the rates applicable to borrowings (debt) outstanding during the accounting period. Sea Manufacturing only had one borrowing, at a rate of 8%. Capitalized interest is computed as follows: Average accumulated expenditures: $1,000,000 × 12/12 = $1,000,000Capitalization rate 8%Capitalized interest expense $ 80,000

Abbott Co. is preparing its statement of cash flows for the year. Abbott's cash disbursements during the year included the following: Payment of interest on bonds payable $500,000Payment of dividends to stockholders 300,000Payment to acquire 1,000 shares ofMarks Co. common stock 100,000 What should Abbott report as total cash outflows for financing activities in its statement of cash flows? $0 $300,000 $800,000 $900,000

Interest payments are included in operating activities. Cash payments to acquire equity instruments are included in investing activities. Only payments of dividends to stockholders are included in financing activities. $300,000

At which of the following amounts should a nongovernmental not-for-profit entity report investments in debt securities? Potential proceeds from liquidation sale Discounted expected future cash flows Quoted market prices Historical cost

Investments in debt securities should be reported at market prices because that is the source of readily available fair value information. Discounted expected future cash flows are required to value financial assets for which there is no market that can provide fair value information. Historical cost is used to value acquisitions of property, plant, and equipment. Liquidation values generally are used when liquidation of an entity is imminent.

Star Co. leases a building for its product showroom. The 10-year nonrenewable lease will expire on December 31, 20X6. In January 20X1, Star redecorated its showroom and made leasehold improvements of $48,000. The estimated useful life of the improvements is 8 years. Star uses the straight-line method of amortization. What amount of leasehold improvements (net of amortization) should Star report in its June 30, 20X1, balance sheet? $45,600 $45,000 $44,000 $43,200

Leasehold improvements on January 1, 20X1 $48,000Amortization of leasehold improvementsJanuary 1 to June 30, 20X1 ($48,000 /6 years left on lease) × 0.5 years - 4,000Leasehold improvements on June 30, 20X1 $44,000======= Remember: Leasehold improvements are capitalized and amortized (as any fixed asset) over the lesser of the useful life of the improvement (8 years) or the remaining lease term (6 years).

On January 1, 20X1, Card Corp. signed a 3-year, noncancelable purchase contract, which allows Card to purchase up to 500,000 units of a computer part annually from Hart Supply Co. at $.10 per unit and guarantees a minimum annual purchase of 100,000 units. During 20X1, the part unexpectedly became obsolete. Card had 250,000 units of this inventory at December 31, 20X1, and believes these parts can be sold as scrap for $.02 per unit. What amount of probable loss from the purchase commitment should Card report in its 20X1 income statement? $24,000 $20,000 $16,000 $8,000

Minimum purchase commitment for 20X2 and 20X3(100,000 units x $.10/u x 2 years) $20,000 Less scrap recovery (100,000 units x $.02 x 2 years) 4,000 Probable loss from purchase commitment $16,000======= Note: The question asks for the probable loss from purchase commitment (i.e., the loss for the remaining two years on the contract). The loss on the 250,000 units already in inventory is not considered part of this loss; it would be reported as an operating loss due to the write-down of inventory due to obsolescence.

On December 1, 20X1, Money Co. gave Home Co. a $200,000, 11% loan. Money paid proceeds of $194,000 after the deduction of a $6,000 nonrefundable loan origination fee. Principal and interest are due in 60 monthly installments of $4,310, beginning January 1, 20X2. The repayments yield an effective interest rate of 11% at a present value of $200,000 and 12.4% at a present value of $194,000. What amount of income from this loan should Money report in its 20X1 income statement? $0 $1,833 $2,005 $7,833

Net proceeds of the loan were $194,000 and the effective interest rate was 12.4%. The journal entry to record the December 31, 20X1, accrual of interest would be: Debit Accrued Interest Receivable 2,005Credit Interest Income 2,005(12.4% × $194,000 × 1/12) The interest income reflects the effective interest rate applied to the net proceeds received. This is an application of the "effective interest" method.

Under FASB Topic 842 lease guidance, which of the following is correct regarding initial direct costs? -Lessees and lessors will apply different definitions of initial direct costs. -Only incremental costs will qualify as initial direct costs. -Lessees and lessors cannot have different initial direct costs for the same lease. -Initial direct costs are expensed for operating leases.

Only incremental costs qualify as initial direct costs subject to capitalization in both lessor and lessee accounting. The same definition is used by both lessors and lessees: these costs are (1) those that the entity would not have incurred if the lease had not been entered into and (2) are the same for both the lessor and lessee. Although the definition of what qualifies as an initial direct cost is the same for the lessee and lessor, the lessee and lessor may have different costs associated with the lease. Initial direct costs are capitalized and amortized over the life of the lease.

Which of the following items is not subject to the application of intraperiod income tax allocation? Discontinued operations Income from continuing operations Prior-period adjustments Gross profit

Operating income is a subtotal well before income tax expense. Income tax expenses during the period are specifically allocated to the other three answer choices (discontinued operations, income from continuing operations, and prior-period adjustments). The following items are subject to the application of intraperiod income tax allocation: Discontinued operations Cumulative effects of accounting changes Prior-period adjustments Direct adjustments to capital accounts Gross profit

On January 1, Year 1, Peabody Co. purchased an investment for $400,000 that represented 30% of Newman Corp.'s outstanding voting stock. For Year 1, Newman reported net income of $60,000 and paid dividends of $20,000. At year-end, the fair value of Peabody's investment in Newman was $410,000. Peabody elected the fair value option for this investment. What amount should Peabody recognize in net income for Year 1 attributable to the investment? $6,000 $10,000 $16,000 $18,000

Peabody would recognize $16,000 in Year 1, calculated as follows: Cost $400,000 Income under equity method ($60,000 x 0.30) 18,000 $18,000 Dividends ($20,000 x 0.30) (6,000) Balance using equity method $412,000 Fair value adjustment* (2,000) (2,000) Net income from the investment $16,000 * Fair value adjustment is $412,000 − $410,000. 1. Calculate current investment amount by adding NI share less div share. 2. Calculate difference between current investment less FV of investment. 3. Subtract difference of #2 from NI calc above only.

Ace Co. settled litigation on February 1, 20X2, for an event that occurred during 20X1. An estimated liability was determined as of December 31, 20X1. This estimate was significantly less than the final settlement. The transaction is considered to be material. The financial statements for year-end 20X1 have not been issued. How should the settlement be reported in Ace's year-end 20X1 financial statements? Disclosure only of the settlement Only an accrual of the settlement Neither a disclosure nor an accrual Both a disclosure and an accrual

Post-balance sheet events (subsequent events) should be explained in the notes to the financial statements when the event is significant. FASB ASC 855-10-55-1 contains guidance about the recognition of subsequent events: "An entity must recognize in the financial statements the effects of all subsequent events that provide additional evidence about conditions that existed at the date of the balance sheet, including the estimates inherent in the process of preparing financial statements." The following is an example of a recognized subsequent event listed in FASB ASC 855-10-55-1: "If the events that gave rise to litigation had taken place before the balance sheet date and that litigation is settled, after the balance sheet date but before the financial statements are issued or are available to be issued, for an amount different from the liability recorded in the accounts, then the settlement amount should be considered in estimating the amount of liability recognized in the financial statements at the balance sheet date." Both a disclosure and an accrual

Pica, a nongovernmental not-for-profit entity, received unconditional promises of $100,000 expected to be collected within one year. Pica received $10,000 prior to year-end. Pica anticipates collecting 90% of the contributions and has a June 30 fiscal year-end. What amount should Pica record as contribution revenue as of June 30? $10,000 $80,000 $90,000 $100,000

Under FASB ASC 958-605-25-2, not-for-profit entities must record unconditional promises to give as contributions revenue when the promise is made. Under FASB ASC 958-605-30-6, such contributions may be recorded at net realizable value, or net of any allowance for uncollectible pledges. $90,000

Sanni Co. had $150,000 in cash-basis pretax income for the year. At the current year-end, accounts receivable decreased by $20,000 and accounts payable increased by $16,000 from their previous year-end balances. Compared to the accrual-basis method of accounting, Sanni's cash-basis pretax income is: higher by $4,000. lower by $4,000. higher by $36,000. lower by $36,000.

Relative to accrual basis, a decrease in accounts receivable is an increase in cash because cash must be received to decrease accounts receivable. Relative to accrual basis, an increase in accounts payable is an increase in cash because accounts payable was increased instead of making cash purchases. Decrease in accounts receivable $20,000 Increase in accounts payable 16,000 Total increase in cash-basis income $36,000 higher

According to the FASB conceptual framework, which of the following situations violates the concept of reliability? -Data on segments having the same expected risks and growth rates are reported to analysts estimating future profits. -Financial statements are issued nine months late. -Management reports to stockholders regularly refer to new projects undertaken, but the financial statements never report project results. -Financial statements include property with a carrying amount increased to management's estimate of market value.

Reliability was addressed in SFAC 5.63. Specifically, the FASB noted: "To be reliable, information about an item must be representationally faithful, verifiable, and neutral. To be reliable, information must be sufficiently faithful in its representation of the underlying resource, obligation, or effect of events and sufficiently free of error and bias to be useful to investors, creditors, and others in making decisions." Clearly, values assigned to property based on management's estimate of fair value are not representationally neutral and would not be considered reliable as defined by the SFAC 5. Late financial statements violate timeliness (relevance), not reliability. Note: The term "reliability" is still used in SFAC 5, even though it has changed to "faithful representation" in SFAC 8.3.

A balance arising from the translation or remeasurement of a subsidiary's foreign currency financial statements is reported in the consolidated income statement when the subsidiary's functional currency is: neither the foreign currency nor the U.S. dollar. the U.S. dollar. the foreign currency. both the foreign currency and the U.S. dollar.

Remeasurement or translation gains and losses appear in the income statement when the subsidiary of a U.S. parent company has the U.S. dollar as their functional currency. If the subsidiary had a foreign currency as their functional currency, the remeasured/translated amount would appear on the balance sheet through other comprehensive income, and not in the consolidated income statement.

The Dunstown County general fund received a notice of a federal grant award for an expenditure-driven (reimbursement) grant in the amount of $1,000,000. Included with the notice was an advance of $250,000. During the year, the County incurred $400,000 of qualifying eligible grant expenditures, and no additional money had been received from the grantor. What would be the amount of deferred revenues reported at the end of the year by the general fund? $0 $600,000 $750,000 $150,000

Resources provided before that period of qualifying activity should be recognized as deferred revenues. Since the amount of qualifying expenditures exceeded the amount of the advance, there would be no deferred revenues reported at year-end. $0

Selected information from the accounts of Row Co. on December 31, 20X1, follows: Total income since incorporation $420,000Total cash dividends paid 130,000Total value of property dividendsdistributed 30,000Excess of proceeds over cost oftreasury stock sold, accountedfor using cost method 110,000 In its December 31, 20X1, financial statements, what amount should Row report as retained earnings? $260,000 $290,000 $370,000 $400,000

Retained earnings on December 31, 20X1, would be computed: Total income since incorporation $420,000Less cash dividends $130,000Property dividends 30,000 160,000Retained earnings on December 31, 20X1 $260,000 Note: The excess of proceeds over cost of treasury stock sold would be credited to "additional paid-in capital" under the cost method.

On October 1, 20X6, EriK's A/C, Inc. agrees to manufacture industrial air conditioning units for five Evergreen Apartment complexes. Under the terms of the contract, Evergreen Apartment will pay EriK's A/C a total of $50,000; Evergreen can cancel the contract at any time but must pay EriK for work completed. EriK believes that they could sell the A/C units to another apartment complex and still make a profit even if Evergreen canceled the contract. The contract is expected to last five months, and as of December 31, 20X6, the job is 50% complete. How much revenue should EriK's A/C recognize in 20X6 for this contract? $0 $50,000 $30,000 $25,000

Revenue for long-term contracts is recognized over time when any one of the following criteria is met: the customer simultaneously receives and consumes the benefits provided by the entity's performance as the entity performs; the entity's performance creates or enhances an asset, such as work-in-process (WIP), that the customer controls as the asset is created or enhanced; or the entity's performance does not create an asset with an alternative use to the entity, and the entity has an enforceable right to payment for performance completed to date. This arrangement does not qualify for revenue recognition over time, because the asset the seller is creating has an alternative use to it. Therefore, EriK must wait until completion of the contract before recognizing revenue. $0

Rollins Corporation acquired 75% of the outstanding stock of Schauer Corporation. The purchase price of the acquisition was $3,960,000. The book value of Schauer's net assets was $4,640,000. Schauer had assets whose fair values were greater than their carrying value by the following amounts: Land $120,000, Buildings $280,000. How much goodwill is implied in Rollins' acquisition of Schauer? $300,000 $240,000 $180,000 $80,000

Rollins will record goodwill of $240,000 by following these steps: -First, the implied total fair value of Schauer must be found by taking the purchase price and adjusting for the controlling interest purchased ($3.96M ÷ 75% = $5.28M total fair value). -Now that we have 100% of the fair value, we have to subtract the fair value of the net assets of Schauer. Those can be found by starting with the book value and adjusting for any differences between book and fair value, for a total net asset value of $5,040,000 ($4,640,000 + $120,000 Land + $280,000 Building). -Goodwill is the difference between the total fair value of the consideration paid and total net assets, or $240,000 ($5.28M − $5.04M). Note that when recognizing the purchase, Rollins would recognize the difference between the actual purchase price and the total fair value of the net assets in the noncontrolling interest (NCI) account.

The FASB's conceptual framework explains both financial and physical capital maintenance concepts. Which capital maintenance concept is applied to currently reported net income, and which is applied to comprehensive income? -Physical capital is applied to currently reported net income; financial capital is applied to comprehensive income. -Financial capital is applied to currently reported net income; physical capital is applied to comprehensive income. - Physical capital is applied to both currently reported net income and comprehensive income. -Financial capital is applied to both currently reported net income and comprehensive income.

SFAC 6, Elements of Financial Statements, contains the following definitions: Capital maintenance concept: the recovery of cost; separation of return on capital from return of capital. Financial capital concept: The effects of price changes on assets held and liabilities owed are recognized as "holding gains and losses" and included in return on capital. Physical capital concept: The effect of price changes are recognized as "capital maintenance adjustments" as a separate element of equity and would not be included in return on capital. SFAC 6 continues: "The financial capital concept is the traditional view and is generally the capital maintenance concept in present primary financial statements. Comprehensive income as defined in paragraph 70 is a return on financial capital." Thus, the financial capital maintenance approach is applied to both currently reported net income and comprehensive income. -Financial capital is applied to both currently reported net income and comprehensive income.

The FASB's conceptual framework explains both financial and physical capital maintenance concepts. Which capital maintenance concept is applied to currently reported net income, and which is applied to comprehensive income? -Physical capital is applied to currently reported net income; financial capital is applied to comprehensive income. -Financial capital is applied to currently reported net income; physical capital is applied to comprehensive income. -Physical capital is applied to both currently reported net income and comprehensive income. -Financial capital is applied to both currently reported net income and comprehensive income.

SFAC 6, Elements of Financial Statements, contains the following definitions: Capital maintenance concept: the recovery of cost; separation of return on capital from return of capital. Financial capital concept: The effects of price changes on assets held and liabilities owed are recognized as "holding gains and losses" and included in return on capital. Physical capital concept: The effect of price changes are recognized as "capital maintenance adjustments" as a separate element of equity and would not be included in return on capital. SFAC 6 continues: "The financial capital concept is the traditional view and is generally the capital maintenance concept in present primary financial statements. Comprehensive income as defined in paragraph 70 is a return on financial capital." Thus, the financial capital maintenance approach is applied to both currently reported net income and comprehensive income. -Financial capital is applied to both currently reported net income and comprehensive income.

Contractual asset or liability disclosures identified in Statement of Financial Accounting Concepts 8 (SFAC 8), Chapter 8, include all of the following except: reporting segments. legal terms. degree of nonperformance risk. method used to calculate the cash flow.

SFAC 8 provides a summary of potential additional disclosures for assets and liabilities resulting from financial instruments or other contracts: the contractual or legal terms (e.g., timing of receipts and disbursements), degree of credit or nonperformance risk, potential effect related to inability to pay or perform, and method used to determine the cash flows. Reporting segments are not one of the suggested disclosure items.

West, Inc., acquired 60% of East Co.'s outstanding common stock. West paid $800,000 to acquire the stock. West plans to relocate East's company headquarters, which is expected to cost between $100,000 and $300,000. The present value of the probability-adjusted relocation cost is $240,000. What is West's acquisition cost? $800,000 $900,000 $1,040,000 $1,100,000

The acquisition of the stock would be reported at cost—the amount paid for the stock. Future relocation costs would not be included in the current cost of the stock. Costs expected to be incurred are not liabilities and should not be included in the acquisition cost. $800000

Glade Co. leases computer equipment to customers under sales-type leases. The equipment has no residual value at the end of the lease and the leases do not contain purchase options. Glade wishes to earn 8% interest on a 5-year lease of equipment with a fair value of $323,400. The present value of an annuity due of $1 at 8% for five years is 4.312. What is the total amount of interest revenue that Glade will earn over the life of the lease? $51,600 $75,000 $129,360 $139,450

The amount of interest revenue that Glade will earn over the life of the lease is $51,600: Annual lease payment = Fair value of equipment / Present value factor= $323,400 / 4.312= $75,000Total lease amount collected = Annual lease payment x 5 years= $75,000 x 5= $375,000Interest revenue = Lease amount collected - Fair value of equipmentearned = $375,000 - $323,400= $51,600

West, Inc., made the following expenditures relating to Product Y: Legal costs to file a patent on Product Y—$10,000. Production of the finished product would not have been undertaken without the patent. Special equipment to be used solely for development of Product Y—$60,000. The equipment has no other use and has an estimated useful life of four years. Labor and material costs incurred in producing a prototype model—$200,000. Cost of testing the prototype—$80,000. What is the total amount of costs that will be expensed when incurred? $280,000 $295,000 $340,000 $350,000

The $10,000 patent cost will be capitalized as an intangible asset in accordance with the provisions of FASB ASC 350-30-25-1. The remaining costs are considered to be research and development costs, as defined in FASB ASC 730-10-55-1. Special developmental equipment and prototypes are addressed in FASB ASC 730-10-55-1. They are considered to be research and development costs which "shall be charged to expense when incurred." Thus the entire $340,000 ($60,000 + $200,000 + $80,000) would be expensed.

In order to remain economically stable, Bradford County believes it needs to maintain the many trails in the county that attract tourists to come and visit. A Trail Preservation Coalition was created by the county and organized as a separate legal entity. The Coalition is governed by a three-person board appointed for 4-year staggered terms by the county commission. The Coalition uses the proceeds of its tax-exempt bonds to finance the maintenance or construction of new trails for the county only. The Coalition owns a building it leases to the county. The bonds are secured by the lease agreement with the county and will be retired through lease payments from the county. Which of the following statements is correct? -The Trail Preservation Coalition is a component unit and will be reported as discretely presented in the county's financial statements. -The Trail Preservation Coalition is a component unit and will be reported as blended with the county's funds. -The Trail Preservation Coalition is a component unit and the county will make a note disclosure only. -The Trail Preservation Coalition is not a component unit of the county and will not be reported by the county.

The Coalition is a component unit of the county. The Coalition's governing body is appointed by the county and the lease constitutes the imposition of a financial burden on the county. A component unit should be included in the reporting entity or primary government's financial statements using the blending method if the component unit's governing body is the same as the primary government; if the component unit provides services entirely or almost entirely to the primary government; or if the component unit's debt is expected to be repaid with resources provided by the primary government even if provided through a lease arrangement. The Coalition will be reported using the blending method because the Coalition provides services entirely to the county and its debt will be repaid with county resources over time. Only one of the three criteria needs to be met for the blending method to be used. In this case, two of the three criteria were met. -The Trail Preservation Coalition is a component unit and will be reported as blended with the county's funds.

Statement of Financial Accounting Concepts 8 (SFAC 8), Chapter 8, lists four limitations/constraints to consider related to disclosure requirements. They are: -cost constraint, materiality, potential adverse consequences, and historical cost. -relevance, representational faithfulness, materiality, and predictive value. -cost constraint, potential adverse consequences, future-oriented information, and relevance. - representational faithfulness, materiality, future-oriented information, and predictive value.

The FASB considers four constraints/limitations related to required financial statement note information: Relevance: Disclosure is based upon relevance, not entity-specific materiality. Cost constraint: The FASB has an expectation that financial statement users have awareness of accounting rules, policies, and regulations; thus, common knowledge can be excluded from the notes. Disclosure should include details of measurement if alternatives exist, methods not obvious to the user, or methods if changed since prior reporting. Potential adverse consequences: The FASB will consider potential adverse consequences. Disclosure can have both beneficial and adverse consequences. Future-oriented information: The FASB does not require entities to disclose predictions of future outcomes that could result in negative consequences. However, two types of forward-looking information are useful and should be provided: (1) estimates and assumptions, and (2) management's existing plans and strategies for management-controlled matters. cost constraint, potential adverse consequences, future-oriented information, and relevance.

Which of the following should be disclosed in a company's financial statements related to deferred taxes? The types and amounts of existing temporary differences The types and amounts of existing permanent differences The nature and amount of each type of operating loss and tax credit carryforward I and II only I and III only II and III only I, II, and III

The FASB requires that a public entity shall disclose the approximate tax effect of each type of temporary difference and carryforward that gives rise to a significant portion of deferred tax liabilities and deferred tax assets (before allocation of valuation allowances). A nonpublic entity shall disclose the types of significant temporary differences and carryforwards but may omit disclosure of the tax effects of each type. An entity shall disclose the amounts and expiration dates of operating loss and tax credit carryforwards for tax purposes. There is no requirement to disclose permanent differences. I and III only

Hoyt Corp.'s current balance sheet reports the following stockholders' equity: 5% cumulative preferred stock, par value $100 per share;2,500 shares issued and outstanding $250,000Common stock, par value $3.50 per share;100,000 shares issued and outstanding 350,000Additional paid-in capital in excess of par value of common stock 125,000Retained earnings 300,000 Dividends in arrears on the preferred stock amount to $25,000. If Hoyt were to be liquidated, the preferred stockholders would receive par value plus a premium of $50,000. The book value per share of common stock is: $7.75. $7.50. $7.25. $7.00.

The book value of common stock is $7.00/share, calculated as follows: Book value/sh = Total Equity - Liquidation value to preferredNo. shares common stock outstanding= $1,025,000(a) - $325,000(b) 100,000 shares= $700,000 100,000 shares= $7.00/share a Total SHE = $250,000 + $350,000 + $125,000 + $300,000 = $1,025,000 b Preferred value = Par + Premiums + Dividends in arrears = $250,000 + $50,000 + $25,000 = $325,000

Grid Corp. acquired some of its own common shares at a price greater than both their par value and original issue price but less than their book value. Grid uses the cost method of accounting for treasury stock. What is the impact of this acquisition on total stockholders' equity and the book value per common share? Increase in both total stockholders' equity and book value per share Increase in total stockholders' equity and decrease in book value per share Decrease in total stockholders' equity and increase in book value per share Decrease in both total stockholders' equity and book value per share

The entry to record the treasury stock is: Dr. Cr.Treasury stock XXXCash XXX Treasury stock is a deduction from total stockholders' equity; therefore total stockholders' equity decreases. Since the price paid for the reacquired shares is less than their book value, the book value per share of the remaining outstanding shares must increase. Before the reacquisition of the shares, the book value per share is the same for all shares (i.e., those that will be reacquired and those that will not be). The reduction in total book value of the entity is equal to the total cost of the shares reacquired, which is less than the total book value of those shares. Thus, the percentage reduction in total book value was less than the percentage reduction in the number of outstanding shares. Therefore, the book value per share of the remaining shares must increase. For example, assume that an entity has total stockholders' equity of $1,000,000 and 10,000 shares outstanding. The book value is $1,000,000 ÷ 10,000 shares = $100. If 1,000 shares are reacquired at $60 per share (an amount less than the $100 book value per share), the entry would be: Dr. Cr.Treasury stock (1,000 x $60) 60,000Cash 60,000 After the reacquisition of the shares, total stockholders' equity is $1,000,000 - $60,000, or $940,000. The book value per share of the remaining outstanding shares is $940,000 ÷ 9,000 outstanding shares, or $104.44. Therefore, the reacquisition of outstanding shares at a price less than their book value per share causes the book value per share of the remaining outstanding shares to increase. Decrease in total stockholders' equity and increase in book value per share

On December 29, Year 1, Action Corp. signed a 7-year finance lease for an airplane to transport its sports team around the country. The airplane's fair value was $841,500. Action made the first annual lease payment of $153,000 on December 31, Year 1. Action's incremental borrowing rate was 12%, and the interest rate implicit in the lease, which was known by Action, was 9%. The following are the rounded present value factors for an annuity due: 9% for 7 years: 5.5 12% for 7 years: 5.1 What amount should Action report as finance lease liability in its December 31, Year 1, balance sheet? $841,500 $780,300 $688,500 $627,300

The finance lease obligation that a lessee recognizes on their books is based on the present value of the lease payments. The lease payments are the annual lease payments made at the beginning of each year of $153,000. The discount rate to get the present value of the payments is the rate implicit in the lease, unless that cannot be readily determined. In the case the lessee cannot determine the implicit rate, the lessee is required to use its incremental borrowing rate. Here, the implicit rate in the lease is known to the lessee, which is 9%. Thus, the amount of the lease liability will be based on the present value factor for an annuity due of 7 years based on 9% (5.5) multiplied by the annual rental of $153,000: 5.5 × $153,000 = $841,500 However, since the first payment was just made, that amount must be lowered to get the remaining liability at the end of the year: $841,500 − $153,000 = $688,500

On June 10, a company issued 2,000 $1,000 5% bonds, payable in 10 years. Each bond contained a detachable warrant that provided a right to purchase 5 shares of $1 par common stock for $30. The value of the warrants at issuance was $50 each. On June 30, the market rate of interest was 9%. At the time of issuance, what amount was the increase in shareholders' equity? $60,000 $100,000 $200,000 $300,000

The issuance of the bond has no impact on stockholders' equity as the bond issuance would increase cash and increase liabilities. Recording the detachable warrants associated with the bond issuance would increase stockholders' equity by their fair value at issuance, $100,000 ($50 × 2,000 warrants).

Which of the following statements about nonlease components is incorrect? -If the component cost represents a transfer of a good or service to the lessee, it is treated as a nonlease component. -The lessee includes the nonlease component when computing the present value of the lease payments. -Nonlease components are expensed by the lessee. -The lessee allocates the consideration to lease and nonlease components on a relative standalone price basis.

The lessee excludes (not includes) the nonlease component when computing the present value of the lease payments. The other answer choices are correct: If the component cost represents a transfer of a good or service to the lessee, it is treated as a nonlease component. Nonlease components are expensed by the lessee. The lessee allocates the consideration to lease and nonlease components on a relative standalone price basis.

Town City has one capital projects fund with assets of $3,000,000, liabilities of $400,000, and outstanding encumbrances of $2,000,000. On the balance sheet prepared at the end of the year, the fund balance would be displayed as: Fund balance—restricted, $2,000,000; Fund balance—unrestricted, $600,000. Fund balance—nonspendable, $2,000,000; Fund balance—unreserved, $600,000. Fund balance—committed, $2,000,000; Fund balance—unassigned, $600,000. Fund balance—committed, $2,000,000; Fund balance—assigned, $600,000.

The outstanding encumbrance would be reflected as either committed or assigned fund balance for $2,000,000, depending on the level of government that entered into the agreement with the vendor. As the vendor in this case is probably a large construction company, it is likely the city council approved the major contract. Nonspendable fund balance would reflect resources such as inventories that cannot be spent, and restricted fund balance would reflect external constraints such as those imposed by creditors or grantors. Only the general fund can have an unassigned fund balance. Any fund balance in excess of the amount of the encumbrance would be considered assigned by being accounted for in a separate governmental fund—a capital projects fund. Fund balance—committed, $2,000,000; Fund balance—assigned, $600,000.

During the current year, Fuqua Steel Co. had the following unusual financial events occur: Bonds payable were retired five years before their scheduled maturity, resulting in a $260,000 gain. Fuqua has frequently retired bonds early when interest rates declined significantly. A steel forming segment suffered $255,000 in losses due to hurricane damage. This was the fourth similar loss sustained in a 5-year period at that location. A segment of Fuqua's operations, steel transportation, was sold at a net loss of $350,000. This was Fuqua's first divestiture of one of its operating segments. Before income taxes, what amount of gain (loss) should be reported separately as a component of income from continuing operations? $260,000 $5,000 $(255,000) $(350,000)

The sale of a segment would be a discontinued operation since its disposition represents a strategic shift. The remaining two items would be reported as a net $5,000 gain component of income from continuing operations.

The net asset reclassifications of a nongovernmental not-for-profit organization would be reported on which of the following? Statement of financial position Statement of activities Statement of cash flows Statement of functional expenses

The statement of activities provides information about the change in amount and nature of net assets by reporting on changes in net assets with donor restrictions and net assets without donor restrictions for a period of time. The statement of financial position is comparable to a balance sheet, and reports assets, liabilities, and net assets (rather than equity) at a point in time. Cash receipts and cash payments are reported on the statement of cash flows.

Spiro Corp. uses the sum-of-the-years'-digits method to depreciate equipment purchased in January 20X1 for $20,000. The estimated salvage value of the equipment is $2,000 and the estimated useful life is four years. What should Spiro report as the asset's carrying amount as of December 31, 20X3? $1,800 $2,000 $3,800 $4,500

The sum-of-the-years'-digits depreciation method entails using a fraction with a numerator equal to the remaining useful life of the asset at the beginning of the year and a denominator equal to the sum of the years' digits. To calculate annual depreciation expense, this fraction is applied each year to the depreciable basis of the asset, which is the historical cost less any salvage value. For an asset with a 4-year estimated useful life, the denominator is 4 + 3 + 2 +1 = 10. The depreciable basis in Spiro's equipment is $20,000 less an estimated salvage value of $2,000 = $18,000. Depreciation expense for 20X1 through 20X3 is calculated as follows: 4/10 × $18,000 = $7,200; 3/10 × $18,000 = $5,400; 2/10 × $18,000 = 3,600. At December 31, 20X3, accumulated depreciation is $16,200 and the cost is $20,000, resulting in a net book value of the equipment of $3,800.

Robbins, Inc., leased a machine from Ready Leasing Co. The lease qualifies as a finance lease and requires 10 annual payments of $10,000 beginning immediately. The lease specifies an interest rate of 12% and a purchase option of $10,000 at the end of the 10th year, even though the machine's estimated value on that date is $20,000. Robbins' incremental borrowing rate is 14%. The present value of an annuity due of $1 at: The present value of $1 at:12% for 10 years is 6.328 12% for 10 years is 0.32214% for 10 years is 5.946 14% for 10 years is 0.270 What amount should Robbins record as lease liability at the beginning of the lease term? $62,160 $64,860 $66,500 $69,720

This is a finance lease, and the lease liability for the lessee at the beginning of the lease is based on the present value of the lease payments, including the purchase at the end of the lease, discounted to their present value (at the beginning of the lease) based on the implicit interest rate specified in the lease itself. Thus, the lease liability here is based on the $10,000 rental payments each year, plus the $10,000 purchase at the end of the 10th year. The present value of both of these items at the beginning of the lease, based on a 12% interest rate would be the $10,000 rent × 6.328 (the present value of an annuity due for 10 periods at 12%) = $63,280. Add this amount to the $10,000 × 0.322 (the present value of $1 at 12%, for 10 periods) = $3,220, and the two amounts add to $66,500.

Yola Co. and Zaro Co. are fuel oil distributors. To facilitate the delivery of oil to their customers, Yola and Zaro exchanged ownership of 1,200 barrels of oil without physically moving the oil. Yola paid Zaro $30,000 to compensate for a difference in the grade of oil. On the date of the exchange, cost and market values of the oil were as follows: Yola Co. Zaro Co.Cost $100,000 $126,000Market values 120,000 150,000 In Zaro's income statement, what amount of gain should be reported from the exchange of the oil? $0 $4,800 $24,000 $30,000

This is a nonmonetary transaction without commercial substance, and thus full gain is not recognized yet, but is instead deferred. Some cash is received, though, so some gain is recognized. $30,000 cash out of a market value of the exchange of $150,000 is 20% of the transaction being in cash, so 20% of the gain is recognized now. Zaro's gain is $150,000 - $126,000, or $24,000, and 20% of $24,000 is $4,800, the gain recognized now.

On July 1, 20X0, Gee, Inc., leased a delivery truck from Marr Corp. under a 3-year operating lease. Total rent for the term of the lease will be $36,000, payable as follows: 12 months at $ 500 = $ 6,00012 months at $ 750 = 9,00012 months at $1,750 = 21,000 All payments were made when due. In Marr's June 30, 20X2, balance sheet, the accrued rent receivable should be reported as: $0. $9,000. $12,000. $21,000.

This problem tests the rule that straight-line recognition should be used to record the rent revenue, regardless of the payment schedule. At $1,000 per month (straight-line), total revenue recognized by the end of the second year should be $24,000. Since cash was actually received at that point in the amount of $15,000 ($9,000 + $6,000), a receivable of $9,000 remains in the rent receivable account for the difference ($24,000 − $15,000). Hint: It helps to do journal entries and T accounts to see how this works, using the following three accounts: rent receivable, rent revenue, and cash.

For a troubled debt restructuring involving only a modification of terms, which of the following items specified by the new terms would be compared to the carrying amount of the debt to determine if the debtor should report a gain on restructuring? The total future cash payments The present value of the debt at the original interest rate The present value of the debt at the modified interest rate The amount of future cash payments designated as principal repayments

This question relates to the debtor's gain on troubled debt restructuring. FASB ASC 310-40-40-1 has changed the treatment of creditor's losses on a restructuring to include the use of present values. Debtor's gains, however, continue to follow FASB ASC 470-60-35-6. Debtor's gains are calculated based on undiscounted amounts. The total future cash payments, including interest, are used to compute the gain on troubled debt restructuring.

A fixed asset with a 5-year estimated useful life and no residual value is sold at the end of the second year of its useful life. How would using the sum-of-the-years'-digits method of depreciation instead of the double-declining-balance method of depreciation affect a gain or loss on the sale of the fixed asset? Gain: decrease; Loss: decrease Gain: decrease; Loss: increase Gain: increase; Loss: decrease Gain: increase; Loss: increase

To answer this question it is best to use an example and see how the numbers work out. If an asset with a 5-year life has a $100,000 cost and a zero salvage value, then the double-declining balance depreciation for the first year would be $40,000: $100,000 × 2/5 (twice straight-line) = $40,000 For the second year, depreciation would be $24,000: $100,000 - $40,000 = $60,000 $60,000 × 2/5 = $24,000 Thus, under double-declining balance, the remaining book value of the asset would be $36,000: $100,000 - $40,000 - $24,000 = $36,000 Under the sum-of-the-years'-digits method, depreciation for the first two years would be 5/15 (5/(5 + 4 + 3 + 2 + 1)) of $100,000, and then 4/15 of $100,000, for a total depreciation for the first two years of 9/15 × $100,000, or $60,000, leaving a book value of $40,000 under the sum-of-the-years'-digits depreciation method. With a higher book value under sum-of-the-years'-digits depreciation at the end of the second year, a sale at a loss would be a higher loss under sum-of-the-years'-digits method, and a sale at a gain would be a lower gain under the sum-of-the-years'-digits method. Gain: decrease; Loss: increase

How should unconditional promises to give received by a nongovernmental not-for-profit entity that will be collected over more than one year be reported? -Long-term contributions receivable, valued at the expected collection amount -Contributions receivable, valued at their present values -Deferred revenue, valued at present value -Contributions receivable, valued at the amount promised

Unconditional contributions receivable expected to be collected over more than one year should be valued using present discounted value techniques and appropriate assumptions. The contributions receivable are valued at present values, not future values. The contributions should be recognized as revenue in the period they are made and not deferred. -Contributions receivable, valued at their present values

Which of the following best describes a situation in which an unconditional contribution should be recognized as revenue by a private not-for-profit organization? -In the period when cash or other assets are received at the carrying value on the books of the donor -In the period received at fair value -In the period in which the donor states its unconditional promise to make the contribution and at the carrying value on the books of the donor -In the period in which the donor states its intention to make the contribution and at fair value

Unconditional contributions, whether promised or received as cash, are recognized as revenue in the period received. Contributions revenue should be measured at fair value, not donor's book value. Donor intentions to give, rather than unconditional promises, are not considered revenue. In the period received at fair value

Sayon Co. issues 200,000 shares of $5 par value common stock to acquire Trask Co. in an acquisition-business combination. The market value of Sayon's common stock is $12. Legal and consulting fees incurred in relationship to the purchase are $110,000. Registration and issuance costs for the common stock are $35,000. What should be recorded in Sayon's additional paid-in capital account for this business combination? $1,545,000 $1,400,000 $1,365,000 $1,255,000

Under FASB ASC 805-20-25, the acquisition will be recorded at fair value: Investment in Trask Co. 2,400,000Common stock ($5 x 200,000) 1,000,000Additional paid-in capital ($7 x 200,000) 1,400,000 Legal and consulting costs are a current expense: Professional services expenses 110,000Cash 110,000 Registration and issuance costs reduce Additional Paid-in Capital: Additional paid-in capital 35,000Cash 35,000 The effect on Additional Paid-in Capital is: Stock issue $1,400,000Registration and issuance costs (35,000)$1,365,000

Gordon, Ltd., a 100%-owned British subsidiary of a U.S. parent company, reports its financial statements in local currency, the British pound. A local newspaper published the following U.S. exchange rates to the British pound at year-end: Current rate $1.50Historical rate (acquisition) 1.70Average rate 1.55Inventory (FIFO) 1.60 Which currency rate should Gordon use to convert its income statement to U.S. dollars at year-end? $1.50 $1.55 $1.60 $1.70

Under FASB ASC 830-30-45-3, all elements of financial statements should be translated by using a current exchange rate. For revenues and expenses, an appropriately weighted-average exchange rate for the period may be used to translate those amounts. 1.55

Tulip Co. owns 100% of Daisy Co.'s outstanding common stock. Tulip's cost of goods sold for the year totals $600,000 and Daisy's cost of goods sold totals $400,000. During the year, Tulip sold inventory costing $60,000 to Daisy for $100,000. By the end of the year, all transferred inventory was sold to third parties. What amount should be reported as cost of goods sold in the consolidated statement of income? $900,000 $940,000 $960,000 $1,000,000

Under the acquisition method, a number of adjusting and eliminating entries are made during the consolidation process. Eliminations may be categorized as those related to the following: The investment account Current-year changes in the investment account Year-end reciprocal balance sheet accounts Reciprocal income statement accounts Intercompany profits and losses Since Daisy sold all of the inventory purchased from Tulip, Daisy would have recognized $100,000 in cost of goods sold (COGS). As Daisy is a 100%-owned subsidiary, 100% of the COGS from Tulip is eliminated (i.e., intercompany profits and losses). Total COGS on the consolidated statement of income is $900,000 ($600,000 + $400,000 − $100,000).

Wall Co. leased office premises to Fox, Inc., for a 5-year term beginning January 2, 20X1. Under the terms of the operating lease, rent for the first year is $8,000 and rent for Years 2 through 5 is $12,500 per annum. However, as an inducement to enter the lease, Wall granted Fox the first six months of the lease rent-free. In its December 31, 20X1, income statement, what amount should Wall report as rental income? $12,000 $11,600 $10,800 $8,000

Wall should report $10,800, calculated as follows: Rent for Year 1 $ 8,000 x 6/12 = $ 4,000 Rent for Years 2-5 $12,500 x 4 = 50,000 Total for five years $54,000======= Rent for 1 year (amount to bereported for 20X1) = $54,000 / 5= $10,800

On January 1, year 1, Boston Group issued $100,000 par value, 5% five-year bonds when the market rate of interest was 8%. Interest is payable annually on December 31. The following present value information is available: 5% 8% Present value of $1 (n = 5) 0.78353 0.68058 Present value of an ordinary annuity (n = 5) 4.32948 3.99271 What amount is the value of net bonds payable at the end of year 1? $88,022 $90,064 $100,000 $110,638

When bonds are sold with a face interest rate (i.e., 5%) less than the market rate (i.e., 8%), they will be sold at a discount, which is calculated as follows: Discounted principal $100,000 × 0.68058 = $ 68,058 Discounted interest payments $100,000 × 5% = $5,000 × 3.99271 = 19,964 Selling price (carrying amount) 88,022 Face amount (100,000) Discount on Bonds Payable $ 11,978========= Interest Amortized Date Cash Expense Discount Carrying Value Jan. 1, year 1 $88,022Dec. 31, year 1 $5,000 $7,042 $2,042 $90,064 $100,000 × 5% $88,022 × 8% ($7,042 - $5,000) ($88,022 + $2,042)

Lily City provides funding for its postemployment benefits other than pension plans primarily on a pay-as-you-go basis. The plan does not meet the criteria specified in GASB Statement 75 to be a trusted "other postemployment benefit" (OPEB) plan. The following information relates to the OPEB plan at the end of the year: Total OPEB liability $10,000,000OPEB plan fiduciary net position 1,000,000Net OPEB liability 9,000,000 What amount should Lily City report as an OPEB liability in its government-wide financial statements? $0 $1,000,000 $9,000,000 $10,000,000

While many governments fund their OPEB plans primarily on a pay-as-you-go basis, they are required to report the true liability for those benefits in their government-wide financial statements. If a government's OPEB plan is administered through a trust that meets specified criteria in GASB Statement 75, the net OPEB liability will be reported. However, if the criteria is not met, the total OPEB liability will be reported ($10,000,000) and any assets set aside for OPEB benefits will continue to be reported as assets of the government.


Ensembles d'études connexes

Conceptual Physics Chapter 4 Review

View Set

Real Estate - Ch 2 - Licensing & Regulation

View Set

CNS MNT GI Disorders Part 2 (mixed upper and lower GI)

View Set

Nutrition Exam 2 Chapter 5 Lipids

View Set

WAP Quiz review Unit 1 and 2 Questions/Ans.

View Set

Psychology 2301 Final Exam Chapters 14-16

View Set

2023-2024 UIL Social Studies Terms List - Individuals

View Set

ATI MedSurg Endocrine Practice Test

View Set

Vocabulary Workshop Level F Unit 6 Antonyms

View Set